Всемирного тяготения: Закон всемирного тяготения Ньютона • Джеймс Трефил, энциклопедия «Двести законов мироздания»

Содержание

Закон всемирного тяготения Ньютона • Джеймс Трефил, энциклопедия «Двести законов мироздания»

Между всеми телами во Вселенной действует сила взаимного притяжения.

На склоне своих дней Исаак Ньютон рассказал, как это произошло: он гулял по яблоневому саду в поместье своих родителей и вдруг увидел луну в дневном небе. И тут же на его глазах с ветки оторвалось и упало на землю яблоко. Поскольку Ньютон в это самое время работал над законами движения (см. Законы механики Ньютона), он уже знал, что яблоко упало под воздействием гравитационного поля Земли. Знал он и о том, что Луна не просто висит в небе, а вращается по орбите вокруг Земли, и, следовательно, на нее воздействует какая-то сила, которая удерживает ее от того, чтобы сорваться с орбиты и улететь по прямой прочь, в открытый космос. Тут ему и пришло в голову, что, возможно, это одна и та же сила заставляет и яблоко падать на землю, и Луну оставаться на околоземной орбите.

Чтобы в полной мере оценить весь блеск этого прозрения, давайте ненадолго вернемся к его предыстории.

Когда великие предшественники Ньютона, в частности Галилей, изучали равноускоренное движение тел, падающих на поверхность Земли, они были уверены, что наблюдают явление чисто земной природы — существующее только недалеко от поверхности нашей планеты. Когда другие ученые, например Иоганн Кеплер (см. Законы Кеплера), изучали движение небесных тел, они полагали что в небесных сферах действуют совсем иные законы движения, нежели законы, управляющие движением здесь, на Земле. История науки свидетельствует, что практически все аргументы, касающиеся движения небесных тел, до Ньютона сводились в основном к тому, что небесные тела, будучи совершенными, движутся по круговым орбитам в силу своего совершенства, поскольку окружность — суть идеальная геометрическая фигура. Таким образом, выражаясь современным языком, считалось, что имеются два типа гравитации, и это представление устойчиво закрепилось в сознании людей того времени. Все считали, что есть земная гравитация, действующая на несовершенной Земле, и есть гравитация небесная, действующая на совершенных небесах.

Прозрение же Ньютона как раз и заключалось в том, что он объединил эти два типа гравитации в своем сознании. С этого исторического момента искусственное и ложное разделение Земли и остальной Вселенной прекратило свое существование.

Результаты ньютоновских расчетов теперь называют законом всемирного тяготения Ньютона. Согласно этому закону между любой парой тел во Вселенной действует сила взаимного притяжения. Как и все физические законы, он облечен в форму математического уравнения. Если M и m — массы двух тел, а D — расстояние между ними, тогда сила F взаимного гравитационного притяжения между ними равна:

= GMm/D2

где G — гравитационная константа, определяемая экспериментально. В единицах СИ ее значение составляет приблизительно 6,67 × 10–11.

Относительно этого закона нужно сделать несколько важных замечаний. Во-первых, его действие в явной форме распространяется на все без исключения физические материальные тела во Вселенной. В частности, сейчас вы и эта книга испытываете равные по величине и противоположные по направлению силы взаимного гравитационного притяжения. Конечно же, эти силы настолько малы, что их не зафиксируют даже самые точные из современных приборов, — но они реально существуют, и их можно рассчитать. Точно так же вы испытываете взаимное притяжение и с далеким квазаром, удаленным от вас на десятки миллиардов световых лет. Опять же, силы этого притяжения слишком малы, чтобы их инструментально зарегистрировать и измерить.

Второй момент заключается в том, что сила притяжения Земли у ее поверхности в равной мере воздействует на все материальные тела, находящиеся в любой точке земного шара. Прямо сейчас на вас действует сила земного притяжения, рассчитываемая по вышеприведенной формуле, и вы ее реально ощущаете как свой вес. Если вы что-нибудь уроните, оно под действием всё той же силы равноускоренно устремится к земле. Галилею первому удалось экспериментально измерить приблизительную величину ускорения свободного падения (см. Уравнения равноускоренного движения) вблизи поверхности Земли. Это ускорение обозначают буквой 

g.

Для Галилея g было просто экспериментально измеряемой константой. По Ньютону же ускорение свободного падения можно вычислить, подставив в формулу закона всемирного тяготения массу Земли M и радиус Земли D, помня при этом, что, согласно второму закону механики Ньютона, сила, действующая на тело, равняется его массе, умноженной на ускорение. Тем самым то, что для Галилея было просто предметом измерения, для Ньютона становится предметом математических расчетов или прогнозов.

Наконец, закон всемирного тяготения объясняет механическое устройство Солнечной системы, и законы Кеплера, описывающие траектории движения планет, могут быть выведены из него. Для Кеплера его законы носили чисто описательный характер — ученый просто обобщил свои наблюдения в математической форме, не подведя под формулы никаких теоретических оснований. В великой же системе мироустройства по Ньютону законы Кеплера становятся прямым следствием универсальных законов механики и закона всемирного тяготения.

То есть мы опять наблюдаем, как эмпирические заключения, полученные на одном уровне, превращаются в строго обоснованные логические выводы при переходе на следующую ступень углубления наших знаний о мире.

Картину устройства солнечной системы, вытекающую из этих уравнений и объединяющую земную и небесную гравитацию, можно понять на простом примере. Предположим, вы стоите у края отвесной скалы, рядом с вами пушка и горка пушечных ядер. Если просто сбросить ядро с края обрыва по вертикали, оно начнет падать вниз отвесно и равноускоренно. Его движение будет описываться законами Ньютона для равноускоренного движения тела с ускорением

g. Если теперь выпустить ядро из пушки в направлении горизонта, оно полетит — и будет падать по дуге. И в этом случае его движение будет описываться законами Ньютона, только теперь они применяются к телу, движущемуся под воздействием силы тяжести и обладающему некой начальной скоростью в горизонтальной плоскости. Теперь, раз за разом заряжая в пушку всё более тяжелое ядро и стреляя, вы обнаружите, что, поскольку каждое следующее ядро вылетает из ствола с большей начальной скоростью, ядра падают всё дальше и дальше от подножия скалы.

Теперь представьте, что вы забили в пушку столько пороха, что скорости ядра хватает, чтобы облететь вокруг земного шара. Если пренебречь сопротивлением воздуха, ядро, облетев вокруг Земли, вернется в исходную точку точно с той же скоростью, с какой оно изначально вылетело из пушки. Что будет дальше, понятно: ядро на этом не остановится и будет и продолжать наматывать круг за кругом вокруг планеты. Иными словами, мы получим

искусственный спутник, обращающийся вокруг Земли по орбите, подобно естественному спутнику — Луне. Так мы поэтапно перешли от описания движения тела, падающего исключительно под воздействием «земной» гравитации (ньютоновского яблока), к описанию движения спутника (Луны) по орбите, не изменяя при этом природы гравитационного воздействия с «земной» на «небесную». Вот это-то прозрение и позволило Ньютону связать воедино считавшиеся до него различными по своей природе две силы гравитационного притяжения.

Остается последний вопрос: правду ли рассказывал на склоне своих дней Ньютон? Действительно ли всё произошло именно так? Никаких документальных свидетельств того, что Ньютон действительно занимался проблемой гравитации в тот период, к которому он сам относит свое открытие, сегодня нет, но документам свойственно теряться.

С другой стороны, общеизвестно, что Ньютон был человеком малоприятным и крайне дотошным во всем, что касалось закрепления за ним приоритетов в науке, и это было бы очень в его характере — затемнить истину, если он вдруг почувствовал, что его научному приоритету хоть что-то угрожает. Датируя это открытие 1666-м годом, в то время как реально ученый сформулировал, записал и опубликовал этот закон лишь в 1687 году, Ньютон, с точки зрения приоритета, выгадал для себя преимущество больше чем в два десятка лет.

Я допускаю, что кого-то из историков от моей версии хватит удар, но на самом деле меня этот вопрос мало беспокоит. Как бы то ни было, яблоко Ньютона остается красивой притчей и блестящей метафорой, описывающей непредсказуемость и таинство творческого познания природы человеком. А является ли этот рассказ исторически достоверным — это уже вопрос вторичный.

См. также:

Закон всемирного тяготения. Сила тяжести.

ЗАКОН ВСЕМИРНОГО ТЯГОТЕНИЯ.

Открыт Ньютоном в 1667 году на основе анализа движения планет (з-ны Кеплера) и, в частности, Луны. В этом же направлении работали Р.Гук (оспаривал приоритет) и Р.Боскович.

 

Все тела взаимодействуют друг с другом с силой, прямо пропорциональной произведению масс этих тел и обратно пропорциональной квадрату расстояния между ними.

Закон справедлив для:

  1. Однородных шаров.
  2. Для материальных точек.
  3. Для концентрических тел.

Гравитационное взаимодействие существенно при больших массах.

Примеры:      

 Притяжение электрона к протону в атоме водорода   » 2×10-11 Н.

 

Тяготение между Землей и Луной» 2×1020 Н.

 

Тяготение между Солнцем и Землей » 3,5×1022 Н.

Применение:

  1. Закономерности движения планет и их спутников. Уточнены законы Кеплера.
  2. Космонавтика. Расчет движения спутников.

Внимание!:

  1. Закон не объясняет причин тяготения, а только устанавливает количественные закономерности.
  2. В случае взаимодействия трех и более тел задачу о движении тел нельзя решить в общем виде. Требуется учитывать «возмущения», вызванные другими телами (открытие Нептуна Адамсом и Леверье в 1846 г. и Плутона в 1930).
  3. В случае тел произвольной формы требуется суммировать взаимодействия между малыми частями каждого тела.

Анализ закона:

  1. Сила направлена вдоль прямой, соединяющей тела.
  2. G — постоянная всемирного тяготения (гравитационная постоянная). Числовое значение зависит от выбора системы единиц.

 

В Международной системе единиц (СИ)        G=6,67.10-11.

G=6,67.10-11

Впервые прямые измерения гравитационной постоянной провел Г. Кавендиш с помощью крутильных весов в 1798 г.

Пусть m1=m2=1 кгR=1 м, тогда: G=F (численно).

Физический смысл гравитационной постоянной:

гравитационная постоянная численно равна модулю силы тяготения, действующей между двумя точечными телами массой по 1 кг каждое, находящимися на расстоянии 1 м друг от друга.

То, что гравитационная постоянная G очень мала показывает, что интенсивность гравитационного взаимодействия мала.

 

СИЛА ТЯЖЕСТИ

Сила тяжести — это сила притяжения тел к Земле (к планете).

 

 

 — из закона Всемирного тяготения. (где — масса планеты, m — масса тела, R — расстояние до центра планеты).

 — сила тяжести из второго закона Ньютона (где m — масса тела, g — ускорение силы тяжести).

  — ускорение силы тяжести не зависит от массы тела (опыты Галилея).

g0=9,81 м/с2 — на поверхности Земли

Если обозначить R0 радиус планеты, а — расстояние до тела от поверхности планеты, то: 

Ускорение силы тяжести зависит:

  1. Массы планеты.
  2. Радиуса планеты.
  3. От высоты над поверхностью планеты.
  4. От географической широты (на полюсах — 9,83 м/с2. на экваторе — 9,79 м/с2.
  5. От залежей полезных ископаемых.

Закон Всемирного Тяготения, гравитация, притяжение, сила, ускорение свободного падения. Направление силы, Ньютон

Тестирование онлайн

  • Основные понятия и определения.

  • Закон всемирного тяготение, ускорение свободного падения.

— По какому закону вы собираетесь меня повесить?
— А мы вешаем всех по одному закону — закону Всемирного Тяготения.

Закон всемирного тяготения

Явление гравитации — это закон всемирного тяготения. Два тела действуют друг на друга с силой, которая обратно пропорциональна квадрату расстояния между ними и прямо пропорциональна произведению их масс.

Математически мы можем выразить этот великий закон формулой

Тяготение действует на огромных расстояниях во Вселенной. Но Ньютон утверждал, что взаимно притягиваются все предметы. А правда ли, что любые два предмета притягивают друг друга? Только представьте, известно, что Земля притягивает вас, сидящих на стуле. Но задумывались ли о том, что компьютер и мышка притягивают друг друга? Или карандаш и ручка, лежащие на столе? В этом случае в формулу подставляем массу ручки, массу карандаша, делим на квадрат расстояния между ними, с учетом гравитационной постоянной, получаем силу их взаимного притяжения. Но, она выйдет на столько маленькой (из-за маленьких масс ручки и карандаша), что мы не ощущаем ее наличие. Другое дело, когда речь идет о Земле и стуле, или Солнце и Земле. Массы значительные, а значит действие силы мы уже можем оценить.

Вспомним об ускорении свободного падения. Это и есть действие закона притяжения. Под действием силы тело изменяет скорость тем медленнее, чем больше масса. В результате, все тела падают на Землю с одинаковым ускорением.

Чем вызвана эта невидимая уникальная сила? На сегодняшний день известно и доказано существование гравитационного поля. Узнать больше о природе гравитационного поля можно в дополнительном материале темы.

Задумайтесь, что такое тяготение? Откуда оно? Что оно собой представляет? Ведь не может быть так, что планета смотрит на Солнце, видит, насколько оно удалено, подсчитывает обратный квадрат расстояния в соответствии с этим законом?

Направление силы притяжения

Есть два тела, пусть тело А и В. Тело А притягивает тело В. Сила, с которой тело А воздействует, начинается на теле B и направлена в сторону тела А. То есть как бы «берет» тело B и тянет к себе. Тело В «проделывает» то же самое с телом А.


Каждое тело притягивается Землей. Земля «берет» тело и тянет к своему центру. Поэтому эта сила всегда будет направлена вертикально вниз, и приложена она с центра тяжести тела, называют ее силой тяжести.

Главное запомнить

1) Закон и формулу;
2) Направление силы тяжести

Некоторые методы геологической разведки, предсказание приливов и в последнее время расчет движения искусственных спутников и межпланетных станций. Заблаговременное вычисление положения планет.

Можем ли мы сами поставить такой опыт, а не гадать, притягиваются ли планеты, предметы?

Такой прямой опыт сделал Кавендиш (Генри Кавендиш (1731-1810) — английский физик и химик) при помощи прибора, который показан на рисунке. Идея состояла в том, чтобы подвесить на очень тонкой кварцевой нити стержень с двумя шарами и затем поднести к ним сбоку два больших свинцовых шара. Притяжение шаров слегка перекрутит нить — слегка, потому что силы притяжения между обычными предметами очень слабы. При помощи такого прибора Кавендишу удалось непосредственно измерить силу, расстояние и величину обеих масс и, таким образом, определить постоянную тяготения G.

Уникальное открытие постоянной тяготения G, которая характеризует гравитационное поле в пространстве, позволила определить массу Земли, Солнца и других небесных тел. Поэтому Кавендиш назвал свой опыт «взвешиванием Земли».

Интересно, что у различных законов физики есть некоторые общие черты. Обратимся к законам электричества (сила Кулона). Электрические силы также обратно пропорциональны квадрату расстояния, но уже между зарядами , и невольно возникает мысль, что в этой закономерности таится глубокий смысл. До сих пор никому не удалось представить тяготение и электричество как два разных проявления одной и той же сущности.

Сила и тут изменяется обратно пропорционально квадрату расстояния, но разница в величине электрических сил и сил тяготения поразительна. Пытаясь установить общую природу тяготения и электричества, мы обнаруживаем такое превосходство электрических сил над силами тяготения, что трудно поверить, будто у тех и у других один и тот же источник. Как можно говорить, что одно действует сильнее другого? Ведь все зависит от того, какова масса и каков заряд. Рассуждая о том, насколько сильно действует тяготение, вы не вправе говорить: «Возьмем массу такой-то величины», потому что вы выбираете ее сами. Но если мы возьмем то, что предлагает нам сама Природа (ее собственные числа и меры, которые не имеют ничего общего с нашими дюймами, годами, с нашими мерами), тогда мы сможем сравнивать. Мы возьмем элементарную заряженную частицу, такую, например, как электрон. Две элементарные частицы, два электрона, за счет электрического заряда отталкивают друг друга с силой, обратно пропорциональной квадрату расстояния между ними, а за счет гравитации притягиваются друг к другу опять-таки с силой, обратно пропорциональной квадрату расстояния.

Вопрос: каково отношение силы тяготения к электрической силе? Тяготение относится к электрическому отталкиванию, как единица к числу с 42 нулями. Это вызывает глубочайшее недоумение. Откуда могло взяться такое огромное число?

Люди ищут этот огромный коэффициент в других явлениях природы. Они перебирают всякие большие числа, а если вам нужно большое число, почему не взять, скажем, отношение диаметра Вселенной к диаметру протона — как ни удивительно, это тоже число с 42 нулями. И вот говорят: может быть, этот коэффициент и равен отношению диаметра протона к диаметру Вселенной? Это интересная мысль, но, поскольку Вселенная постепенно расширяется, должна меняться и постоянная тяготения. Хотя эта гипотеза еще не опровергнута, у нас нет никаких свидетельств в ее пользу. Наоборот, некоторые данные говорят о том, что постоянная тяготения не менялась таким образом. Это громадное число по сей день остается загадкой.

Эйнштейну пришлось видоизменить законы тяготения в соответствии с принципами относительности. Первый из этих принципов гласит, что расстояние х нельзя преодолеть мгновенно, тогда как по теории Ньютона силы действуют мгновенно. Эйнштейну пришлось изменить законы Ньютона. Эти изменения, уточнения очень малы. Одно из них состоит вот в чем: поскольку свет имеет энергию, энергия эквивалентна массе, а все массы притягиваются, — свет тоже притягивается и, значит, проходя мимо Солнца, должен отклоняться. Так оно и происходит на самом деле. Сила тяготения тоже слегка изменена в теории Эйнштейна. Но этого очень незначительного изменения в законе тяготения как раз достаточно, чтобы объяснить некоторые кажущиеся неправильности в движении Меркурия.

Физические явления в микромире подчиняются иным законам, нежели явления в мире больших масштабов. Встает вопрос: как проявляется тяготение в мире малых масштабов? На него ответит квантовая теория гравитации. Но квантовой теории гравитации еще нет. Люди пока не очень преуспели в создании теории тяготения, полностью согласованной с квантовомеханическими принципами и с принципом неопределенности.

ВСЕМИРНОГО ТЯГОТЕНИЯ ЗАКОН • Большая российская энциклопедия

  • В книжной версии

    Том 6. Москва, 2006, стр. 46

  • Скопировать библиографическую ссылку:


Авторы: Е. 2$ раз в про­доль­ном на­прав­ле­нии, т. е. ве­дёт се­бя точ­но так же, как и элек­трич. си­ла при­тя­же­ния ме­ж­ду дву­мя раз­но­имён­ны­ми элек­три­че­ски за­ря­жен­ны­ми те­ла­ми, дви­жу­щи­ми­ся с те­ми же ско­ро­стя­ми. Это на­хо­дит­ся в со­от­вет­ст­вии с Пу­ан­ка­ре прин­ци­пом от­но­си­тель­но­сти. В слу­чае силь­ных гра­ви­та­ци­он­ных по­лей, соз­да­вае­мых мас­сив­ны­ми звёз­да­ми, по­яв­ля­ет­ся бо­лее слож­ная за­ви­си­мость си­лы при­тя­же­ния от рас­стоя­ния.

Закон всемирного тяготения Ньютона

Чувствуете, как вас тянет к другим людям? На самом деле это и правда происходит, согласно закону всемирного тяготения Ньютона.

Признайтесь, вы тоже не до конца поняли, что такое закон всемирного тяготения Ньютона, когда учились в школе? Это неудивительно: человечество, за исключением нескольких астрономов и физиков, даже не подозревало о нем до 1687 года, да и потом еще лет 200 ученые трудились над строгим обоснованием гениальной теории Ньютона. Так что нет ничего стыдного даже для взрослого человека в том, чтобы освежить свои знания о неведомой силе, которая притягивает все тела во Вселенной, определяет траектории движения планет Солнечной системы, создает приливы и отливы и запускает течение рек на Земле, а однажды подсказала ученым сам факт существования планеты Нептун.

Как был открыт закон всемирного тяготения?

По легенде, теория гравитации родилась в голове Ньютона благодаря упавшему на него яблоку, и это не пустой миф. Близкие знакомые ученого оставили свидетельства о разговоре с ним и о самом «яблочном инциденте», который, по-видимому, случился в 1666 году, когда молодой Исаак пережидал эпидемию бубонной чумы в поместье своей матери. Находясь в самоизоляции, 23-летний юноша размышлял о том, почему яблоко падает перпендикулярно к земной поверхности, а не вбок или вверх, и пришел к выводу о том, что яблоко притягивает Землю так же, как Земля притягивает яблоко.

Пока чума косила англичан, погубив пятую часть населения Лондона, научная мысль Ньютона шагала за пределы нашей планеты и он спрашивал себя: как далеко простирается эта незримая сила (гравитация) и не она ли удерживает Луну вблизи Земли, не давая ей улететь? История с падением яблока стала популярна благодаря Вольтеру, описавшему инцидент со слов племянницы Ньютона, и биографу Уильяму Стьюкли, который изложил ее в книге «Воспоминания о жизни Ньютона», выпущенной в 1752 году.

На формулировку закона всемирного тяготения у гениального британского ученого ушло два десятка лет: впервые он оповестил мир о нем в 1687 году — в своем фундаментальном труде «Математические начала натуральной философии». Так наконец удалось дать объяснение траектории движения планет вокруг Солнца, обосновать открытия немецкого астронома Кеплера, сформулированные в начале XVII века, ответив на главный вопрос: почему планеты движутся не по кругу, а по эллиптической орбите? Закон всемирного тяготения Ньютона и сама идея гравитации помогли объяснить феномены, о которых эмпирическим путем уже догадывались самые наблюдательные ученые. Большинство же людей верили в божий промысел, считали Землю центром Вселенной и даже не подозревали о том, что на яблоко и Луну влияют одни и те же физические законы.

Притяженья больше нет?

Если все тела во Вселенной притягиваются, то почему мы чувствуем притяжение только к Земле, а не к холодильнику или друг к другу? Все дело в массе и расстоянии: до тех пор, пока масса предмета мала, а расстояние велико, мы не чувствуем никакого притяжения. И лишь когда речь идет о такой махине, как Земля, мы сполна ощущаем силу тяжести — одну из самых заметных проявлений силы всемирного тяготения.

Закон всемирного тяготения гласит: два любых тела притягиваются друг к другу с силой, прямо пропорциональной массе каждого из них и обратно пропорциональной квадрату расстояния между ними

Для подсчета используется формула: F = G ∙ (m1 ∙ m2) / R², где m — масса, R — расстояние между телами, G — гравитационная постоянная, значение которой было определено экспериментально. Эта постоянная G очень мала (6,67 ∙ 10–11 м³ / (кг ∙ с²)) — именно поэтому сила, с которой притягиваются тела небольшой массы, нами совершенно не ощущается.

Был ли Ньютон первооткрывателем?

С момента публикации «Начал» многим ученым не нравилось, что Ньютон не объяснил физическую природу гравитации, не назвал ее источник, не привел доказательства. Некоторые ученые считали, что ученый промышляет плагиатом: мысль о том, что движение планет объясняется действием силы, которая притягивает каждую планету к Солнцу, уже высказывалась ранее, в том числе английским физиком Робертом Гуком — он даже сформулировал, что эта сила убывает обратно пропорционально квадрату расстояния от Солнца. Свою теорию Гук изложил в том самом 1666 году, когда на Исаака упало яблоко, а в 1679 году посылал Ньютону письмо, где предлагал сотрудничать по решению этой задачи, но получил отказ и заверения о том, что эта тема давно не занимает адресата. В дальнейшем Гук требовал указывать его имя как первого автора закона тяготения и открыто обвинял Ньютона в плагиате. Ученые конфликтовали до конца жизни Гука, а спор о том, кто был первым, продолжался даже в XX веке.

«К сожалению, нам неизвестны детали того логического пути, которым Ньютон пришел к закону всемирного тяготения», — писали американские ученые в книге «Физика» в 1960 году.

«Если связать в одно все предположения и мысли Гука о движении планет и тяготении, высказанные им в течение почти 20 лет, то мы встретим почти все главные выводы «Начал» Ньютона, только высказанные в неуверенной и мало доказательной форме. Не решая задачи, Гук нашел ее ответ», — писал советский ученый Сергей Вавилов. Ньютон был блестящим математиком и смог решить поставленную Гуком задачу.

Ньютон помог открыть Нептун

Лишь после того, как ньютоновская теория стала основой небесной механики в XVIII веке, физики приняли ее более благосклонно. Закон всемирного тяготения Ньютона стал подарком для астрономов, так как математически объяснил почти все, что происходит во Вселенной. Но, пожалуй, главным вкладом Ньютона в астрономию стало открытие в 1846 году Нептуна — самой дальней от Земли планеты и первой, обнаруженной путем математических расчетов.

Этому знаменательному событию предшествовало открытие Урана в 1781 году английским астрономом Уильямом Гершелем. Наблюдавшие за ее движением астрономы многие годы народились в затруднении: реальная орбита Урана не совпадала с вычисленной. Это недоразумение заставляло думать о том, что за Ураном прячется еще одна планета, которая влияет на нее своим притяжением. Французский математик Урбен Леверье провел расчеты с помощью ньютоновой механики и указал астрономам, где именно нужно искать восьмую планету. 2}}\ — ускорение свободного падения на высоте hh от поверхности Земли.

Силой тяжести называют силу, с которой тело притягивается к планете:

\[\boxed{F = mg} — \mathrm{сила}\ \mathrm{тяжести}\]

Рассмотрим твёрдое тело, расположенное на горизонтальной неподвижной опоре: под действием силы тяжести тело деформируется. Если тело находится на опоре, то на нижний слой действуют все верхние слои, и, как следствие, этот слой деформируется наибольшим образом. На предпоследний слой действует меньшее количество слоёв, и он деформируется меньше. Таким образом, тело, бывшее прямоугольным, примет вид трапеции. Нижний слой приблизился при такой деформации к центру тела, а значит, возникла сила упругости, направленная в сторону, противоположную направлению смещения частиц при деформации. Сила упругости, возникшая внутри данного тела, направлена перпендикулярно опоре. Эту силу, созданную деформированным телом и приложенную к опоре, называют весом тела. Опора под действием веса деформируется. Противоположная весу сила упругости действует на данное тело со стороны деформированной опоры и тоже направлена перпендикулярно опоре, но называется силой реакции опоры NN (от слова normal — перпендикуляр).










Рис. 9

На рисунке 9 тело не касается опоры для того, чтобы показать, что вес приложен к опоре, а сила реакции опоры к телу. В действительности площадь реального соприкосновения твёрдых тел невелика. Большей частью между телами находится тонкий слой воздуха.

Вполне очевидно, что если опоры нет, то и веса тело иметь не будет. Такое случится в том случае, если тело движется под действием только одной силы – силы тяготения.

Невесомостью называют состояние тела, когда оно движется под действием только силы тяготения.

Так же легко понять, что если на тело действует две силы (сила тяжести и сила реакции опоры), то эти силы не обязательно равны друг другу. Одна из них может быть больше другой.

Рассмотрим движение тела, помещённого в лифт. Пусть сам лифт движется с ускорением a→\vec a. Такое ускорение будет в двух случаях: 1) лифт поднимается равно ускорено, 2) лифт опускается равнозамедленно. Второй закон Ньютона для данного тела примет вид:








Рис. 10

N→+mg→=ma→.\vec N + m \vec g = m \vec a.

При рассмотрении данного движения из лабораторной неподвижной системы отсчёта OyOy увидим, что в проекции на вертикальную ось OyOy второй закон запишется следующим образом:

N-mg=ma,N — mg = ma,

откуда 

N=ma+mg=m(g+a).N = ma + mg = m(g+a).

Но по третьему закону Ньютона знаем, что сила реакции опоры и вес тела равны и противоположны, следовательно:

N=P,N = P,

тогда: P=m(g+a) -\boxed{P = m(g+a)}\ — вес тела, движущегося с ускорением, направленным вверх (рис. 10). 2}{R})}\ — вес тела, движущегося с ускорением, направленным вверх (вогнутая дорога).

Важное дополнение:

Для рассматриваемой силы, называемой весом, важно понимать и уметь правильно изображать точку приложения этой силы.








На рисунке 11а показан лифт, у которого нет ускорения. Тогда сила тяжести равна силе реакции опоры . А по третьему закону Ньютона, сила реакции опоры равна весу тела. Точка приложения силы тяжести расположена в геометрическом центре тела, если тело однородно и правильной формы. Точка приложения силы реакции опоры должна быть изображена внутри тела вблизи с нижней поверхностью тела на линии действия силы тяжести. Последнее свойство на рисунке не выдержано для удобства изображения (иначе силы на рисунке будут накладываться друг на друга). Точка приложения веса тела находится внутри опоры (пола лифта) вблизи поверхности на линии действия силы реакции опоры.










                Рис. 2} = \frac 18 F = 10\ \text{Н}.\]

Сила притяжения шаров станет меньше на 10 Н10\ \text{Н}, следовательно, станет равной 70 Н70\ \text{Н}.

Для проверки закона всемирного тяготения измерили рекордно слабое гравитационное поле — Наука

ТАСС, 10 марта. Ученые провели самую точную проверку закона всемирного тяготения Ньютона. Для этого они измерили рекордно слабое гравитационное поле, которое образовали две микроскопических золотых сферы массой в 90 мг и радиусом в 1 мм. Описание эксперимента опубликовал научный журнал Nature.

На эту тему

«Ранее подобные измерения проводили для объектов массой в килограмм и более, а мы сделали это со сферами массой в 90 мг. Это открывает дорогу для еще более точных измерений – например, для тел, сопоставимых по массе с самыми крупными элементарными частицами», –пишут исследователи.

Физиков давно интересует, работает ли классический закон всемирного тяготения на микроскопических масштабах. Проверить это сложно, поскольку хотя гравитация действует на очень большие расстояния, но при этом ее тяжело измерить. Поэтому погрешности в подобных измерениях могут возникать даже из-за самых слабых помех.

Однако подобные эксперименты очень важны для физиков, так как благодаря им можно проверить, существуют ли скрытые измерения пространства и какими свойствами обладают темная материя и энергия.

Большой шаг в эту сторону сделали физики под руководством профессора Венского университета Маркуса Аспельмейера. Они создали установку, с помощью которой можно с максимальной точностью определить, с какой силой притягивают друг друга сферы из золота диаметром всего в миллиметр и массой в 90 миллиграмм.

Установка выглядит как сделанное из сверхтонкой проволоки маленькое коромысло. К его середине прикреплено зеркало, а также нить, на которой вся конструкция может вращаться вокруг своей оси. К концам коромысла прикрепляются объекты, гравитационные свойства которых физики пытаются изучить.

На эту тему

Зеркало постоянно подсвечивает луч лазера, отражение которого улавливает фотодетектор. Благодаря этому на то, в каком направлении будет отражаться луч света, влияют даже малейшие сдвиги в положении сфер и прикрепленного к ним «коромысла».

С помощью этой установки Аспельмейер и его коллеги проследили, как по мере удаления и сближения шариков друг с другом их притягивала еще одна золотая сфера, которую ученые поместили на небольшом расстоянии от одного из шариков. Результаты измерений физики использовали для вычисления гравитационной постоянной G, значение которой определено не максимально точно.

Из-за того, что гравитационные взаимодействия между сферами очень малы (они не превышают 0,09 пиконьютонов), ученые повторяли эксперимент более 350 раз, чтобы исключить влияние всех источников помех. В итоге авторы эксперимента получили относительно точное значение G, которое расходится с общепринятыми оценками примерно на 9%.

Ученые надеются, что дальнейшие измерения с еще более легкими объектами помогут проверить, было ли это расхождение простой погрешностью измерений или же оно существует на самом деле. Кроме того, эти опыты дадут физикам шанс вплотную приблизиться к тому пределу масс, где теоретически должны действовать квантовые гравитационные эффекты. Благодаря этому можно будет проверить теории, которые допускают их существование.

Закон всемирного тяготения Ньютона

Как обсуждалось ранее в Уроке 3, Исаак Ньютон сравнил ускорение Луны с ускорением объектов на Земле. Полагая, что за каждую из них ответственны гравитационные силы, Ньютон смог сделать важный вывод о зависимости силы тяжести от расстояния. Это сравнение привело его к выводу, что сила гравитационного притяжения между Землей и другими объектами обратно пропорциональна расстоянию, отделяющему центр Земли от центра объекта.Но расстояние — не единственная переменная, влияющая на величину гравитационной силы. Рассмотрим знаменитое уравнение Ньютона

. F net = m • a

Ньютон знал, что сила, вызывающая ускорение яблока (гравитация), должна зависеть от массы яблока. А поскольку сила, вызывающая ускорение яблока вниз, также вызывает ускорение земли вверх (третий закон Ньютона), эта сила также должна зависеть от массы Земли.Таким образом, для Ньютона сила тяжести, действующая между Землей и любым другим объектом, прямо пропорциональна массе Земли, прямо пропорциональна массе объекта и обратно пропорциональна квадрату расстояния, разделяющего центры объектов. земля и объект.

УНИВЕРСАЛЬНОЕ УРАВНЕНИЕ ГРАВИТАЦИИ

Но закон всемирного тяготения Ньютона распространяет гравитацию за пределы Земли. Закон всемирного тяготения Ньютона говорит о универсальности гравитации.Место Ньютона в Зале славы гравитации связано не с его открытием гравитации, а с его открытием универсальности гравитации. ВСЕ объектов притягивают друг друга силой гравитационного притяжения. Гравитация универсальна. Эта сила гравитационного притяжения напрямую зависит от масс обоих объектов и обратно пропорциональна квадрату расстояния, разделяющего их центры. Вывод Ньютона о величине гравитационных сил символически резюмируется как

.

Поскольку гравитационная сила прямо пропорциональна массе обоих взаимодействующих объектов, более массивные объекты будут притягивать друг друга с большей силой гравитации.Таким образом, когда масса любого объекта увеличивается, сила гравитационного притяжения между ними также увеличивается. Если масса одного из предметов увеличивается вдвое, то сила тяжести между ними увеличивается вдвое. Если масса одного из предметов увеличивается втрое, то сила тяжести между ними увеличивается втрое. Если масса обоих объектов увеличивается вдвое, то сила тяжести между ними увеличивается в четыре раза; и так далее.

Поскольку сила тяжести обратно пропорциональна квадрату расстояния разделения между двумя взаимодействующими объектами, большее расстояние разделения приведет к более слабым силам гравитации.Так как два объекта отделены друг от друга, сила гравитационного притяжения между ними также уменьшается. Если расстояние между двумя объектами увеличивается вдвое (увеличивается в 2 раза), то сила гравитационного притяжения уменьшается в 4 раза (2 во второй степени). Если расстояние между любыми двумя объектами увеличивается в три раза (увеличивается в 3 раза), то сила гравитационного притяжения уменьшается в 9 раз (3 во второй степени).

Пропорциональное мышление об уравнении Ньютона

Пропорциональности, выраженные универсальным законом всемирного тяготения Ньютона, графически представлены на следующем рисунке. Обратите внимание, как сила тяжести прямо пропорциональна произведению двух масс и обратно пропорциональна квадрату расстояния разделения.

Еще один способ представления пропорциональностей — это выражение отношений в форме уравнения с использованием константы пропорциональности.Это уравнение показано ниже.

Константа пропорциональности (G) в приведенном выше уравнении известна как универсальная гравитационная постоянная . Точное значение G было экспериментально определено Генри Кавендишем через столетие после смерти Ньютона. (Этот эксперимент будет обсуждаться позже в Уроке 3.) Значение G составляет

. G = 6,673 x 10 -11 Н м 2 / кг 2

Блоки на G могут показаться довольно странными; тем не менее они разумны.Когда единицы на G подставляются в приведенное выше уравнение и умножаются на м 1 • м 2 единиц и делятся на d 2 единиц, результатом будет Ньютоны — единица силы.

Использование уравнения тяготения Ньютона для решения задач

Знание значения G позволяет нам вычислить силу гравитационного притяжения между любыми двумя объектами известной массы и известного расстояния разделения.В качестве первого примера рассмотрим следующую проблему.

Пример задачи № 1

Определите силу гравитационного притяжения между Землей (m = 5,98 x 10 24 кг) и студентом-физиком весом 70 кг, если студент стоит на уровне моря, на расстоянии 6,38 x 10 6 м от центра Земли. .

Решение задачи заключается в подстановке известных значений G (6.673 x 10 -11 Н м 2 / кг 2 ), м 1 (5,98 x 10 24 кг), м 2 (70 кг) и d (6,38 x 10 6 м) в универсальное уравнение гравитации и решение для F grav . Решение следующее:


Пример задачи № 2

Определите силу гравитационного притяжения между землей (m = 5.98 x 10 24 кг) и студент-физик весом 70 кг, если студент находится в самолете на высоте 40000 футов над земной поверхностью. Это поместит студента на расстояние 6,39 x 10 6 м от центра Земли.

Решение задачи заключается в подстановке известных значений G (6,673 x 10 -11 Н · м 2 / кг 2 ), м 1 (5,98 x 10 24 кг), м 2 (70 кг) и d (6. 39 x 10 6 м) в универсальное уравнение гравитации и решение для F grav . Решение следующее:

Можно сделать два общих концептуальных комментария к результатам двух приведенных выше расчетов. Во-первых, обратите внимание на то, что сила тяжести, действующая на ученика (также известная как вес ученика), меньше на самолете на высоте 40000 футов, чем на уровне моря. Это иллюстрирует обратную зависимость между разделительным расстоянием и силой тяжести (или, в данном случае, весом ученика).На большей высоте ученик весит меньше. Однако простое изменение на 40 000 футов от центра Земли практически незначительно. Это изменение высоты изменило вес ученика на 2 Н, что намного меньше 1% от первоначального веса. Расстояние в 40000 футов (от поверхности земли до высотного самолета) не так уж и далеко по сравнению с расстоянием 6,38 x 10 6 м (что эквивалентно почти 20000000 футов от центра Земли до поверхность земли).Это изменение расстояния похоже на капля в ведре по сравнению с большим радиусом Земли. Как показано на диаграмме ниже, расстояние разделения становится гораздо более важным при значительном изменении.

Второй концептуальный комментарий к приведенным выше примерным расчетам заключается в том, что использование универсального уравнения гравитации Ньютона для вычисления силы тяжести (или веса) дает тот же результат, что и при его вычислении с использованием уравнения, представленного в Блоке 2:

F grav = m • g = (70 кг) • (9.8 м / с 2 ) = 686 Н

Оба уравнения дают один и тот же результат, потому что (как мы изучим позже в Уроке 3) значение g эквивалентно отношению (G • M земля ) / (R земля ) 2 .

Универсальность гравитации

Гравитационные взаимодействия существуют не просто между Землей и другими объектами; и не просто между Солнцем и другими планетами.Гравитационные взаимодействия существуют между всеми объектами с интенсивностью, которая прямо пропорциональна произведению их масс. Итак, когда вы сидите на своем месте в классе физики, вас притягивает гравитационное влечение к партнеру по лаборатории, к столу, за которым вы работаете, и даже к своей книге по физике. Революционная идея Ньютона заключалась в том, что гравитация универсальна — ВСЕ объекты притягиваются пропорционально произведению их масс. Гравитация универсальна. Конечно, большинство гравитационных сил настолько минимальны, чтобы их можно было заметить.Гравитационные силы распознаются только тогда, когда массы объектов становятся большими. Чтобы проиллюстрировать это, используйте универсальное уравнение тяготения Ньютона для вычисления силы тяжести между следующими знакомыми объектами. Нажмите кнопки, чтобы проверить ответы.

Масса объекта 1

(кг)

Масса объекта 2

(кг)

Расстояние разделения

(м)

Сила тяжести

(н. )

а.

Футболист

100 кг

Земля

5,98 x10 24 кг

6,38 x 10 6 м

(на поверхности)

г.

Балерина

40 кг

Земля

5.98 x10 24 кг

6,38 x 10 6 м

(на поверхности)

г.

Студент-физик

70 кг

Земля

5,98 x10 24 кг

6.60 x 10 6 м

(малая орбита)

г.

Студент-физик

70 кг

Студент-физик

70 кг

1 метр

e.

Студент-физик

70 кг

Студент-физик

70 кг

0,2 ​​м

ф.

Студент-физик

70 кг

Книга по физике

1 кг

1 метр

грамм. Студент-физик

70 кг

Луна

7,34 x 10 22 кг

1,71 x 10 6 м

(на поверхности)

час Студент-физик

70 кг

Юпитер

1. 901 x 10 27 кг

6,98 x 10 7 м

(на поверхности)

Сегодня закон всемирного тяготения Ньютона является широко принятой теорией. Он направляет усилия ученых при изучении планетных орбит. Зная, что все объекты оказывают гравитационное влияние друг на друга, небольшие возмущения в эллиптическом движении планеты можно легко объяснить.По мере того, как планета Юпитер приближается к планете Сатурн по своей орбите, она имеет тенденцию отклоняться от своего плавного пути; это отклонение, или возмущение , легко объяснить, если учесть влияние гравитационного притяжения между Сатурном и Юпитером. Сравнение Ньютоном ускорения яблока с ускорением луны привело к удивительно простому выводу о природе гравитации, которая пронизывает всю Вселенную. Все объекты притягивают друг друга с силой, которая прямо пропорциональна произведению их масс и обратно пропорциональна расстоянию между ними.

Расследовать! Используйте приведенный ниже виджет Закон всемирного тяготения Ньютона , чтобы исследовать влияние масс объекта и расстояния разделения на величину гравитационного притяжения. Введите массы двух объектов и расстояние до них. Затем нажмите кнопку Submit , чтобы увидеть гравитационную силу. Поэкспериментируйте с различными значениями массы и расстояния.

Как Ньютон установил, что именно сила тяжести между Солнцем и планетами является силой, обеспечивающей движение планет по их эллиптическому пути? Щелкните, чтобы увидеть.


Мы хотели бы предложить … Иногда просто прочитать об этом недостаточно. Вы должны с ним взаимодействовать! И это именно то, что вы делаете, когда используете один из интерактивных материалов The Physics Classroom. Мы хотели бы предложить вам совместить чтение этой страницы с использованием нашего Gravitation Interactive. Вы можете найти его в разделе Physics Interactives на нашем сайте. Gravitation Interactive позволяет учащемуся интерактивно исследовать закон обратных квадратов гравитации.

Проверьте свое понимание

1. Предположим, что два объекта притягиваются друг к другу с силой тяжести 16 единиц.Если расстояние между двумя объектами удвоится, какова новая сила притяжения между двумя объектами?

2. Предположим, что два объекта притягиваются друг к другу с силой тяжести 16 единиц. Если расстояние между двумя объектами уменьшится вдвое, то какова новая сила притяжения между двумя объектами?

3.Предположим, что два объекта притягиваются друг к другу с силой тяжести 16 единиц. Если бы масса обоих объектов увеличилась вдвое, и если бы расстояние между объектами осталось прежним, то какой была бы новая сила притяжения между двумя объектами?

4. Предположим, что два объекта притягиваются друг к другу с силой тяжести 16 единиц. Если бы масса обоих объектов была удвоена, и если бы расстояние между объектами было удвоено, то какой была бы новая сила притяжения между двумя объектами?


5.Предположим, что два объекта притягиваются друг к другу с силой тяжести 16 единиц. Если бы масса обоих объектов была утроена, и если бы расстояние между объектами было удвоено, то какой была бы новая сила притяжения между двумя объектами?

6. Предположим, что два объекта притягиваются друг к другу с силой тяжести 16 единиц. Если бы масса объекта 1 была удвоена, и если бы расстояние между объектами было утроено, то какой была бы новая сила притяжения между двумя объектами?


7.Считается, что с возрастом звезда претерпевает множество изменений. Один из последних этапов жизни звезды — это гравитационный коллапс в черную дыру. Что будет с орбитой планет солнечной системы, если наша звезда (Солнце превратится в черную дыру)? (И, конечно, это предполагает, что планеты не подвержены влиянию предыдущих стадий развития Солнца.)


8. Недавно завершив свой первый курс физики, Дон Велл разработала новый бизнес-план на основе темы «Физика для лучшей жизни » ее учителя.Доун узнала, что объекты весят разное количество на разном расстоянии от центра Земли. Ее план включает покупку золота на вес на одной высоте, а затем продажу его на другой высоте по той же цене за вес. Следует ли Dawn покупать на большой высоте и продавать на низкой или наоборот?


9. Фред очень беспокоится о своем весе, но редко что-то с ним делает. Узнав о законе всемирного тяготения Ньютона на уроке физики, он начинает беспокоиться о возможном влиянии изменения массы Земли на его вес.Во время (редкого) свободного времени за обеденным столом он говорит: «Как бы изменился мой вес, если бы масса Земли увеличилась на 10%?» Как бы вы ответили Фреду?

10. При сравнении данных о массе и размере планет Земля и Юпитер было замечено, что Юпитер примерно в 300 раз массивнее Земли. Можно быстро заключить, что объект на поверхности Юпитера и будет весить в 300 раз больше, чем на поверхности Земли.Например, можно было ожидать, что человек, который весит 500 Н на Земле, будет весить 150000 Н на поверхности Юпитера. Но это не так. Фактически, человек массой 500 N на Земле весит около 1500 N на поверхности Юпитера. Объясните, как это может быть.

Закон всемирного тяготения Ньютона

Закон всемирного тяготения

Объекты с массой ощущают силу притяжения, которая пропорциональна их массе и обратно пропорциональна квадрату расстояния.

Цели обучения

Выразите закон всемирного тяготения в математической форме

Основные выводы

Ключевые моменты
  • Закон всемирного тяготения сэра Исаака Ньютона вдохновил падение яблока с дерева. 2} [/ latex] где [latex] \ text {G} [/ latex] — гравитационная постоянная.
Ключевые термины
  • индукция : Используйте индуктивные рассуждения для обобщения и интерпретации результатов применения закона тяготения Ньютона.
  • обратный : противоположный по действию, характеру или порядку.

В то время как яблоко могло и не поразить сэра Исаака Ньютона в голову, как предполагает миф, падение одного из них действительно вдохновило Ньютона на одно из величайших открытий в механике: Закон всемирного тяготения . Размышляя о том, почему яблоко никогда не падает вбок, вверх или в любом другом направлении, кроме перпендикулярного земле, Ньютон понял, что сама Земля должна быть ответственна за движение яблока вниз.

Теоретически предполагая, что эта сила должна быть пропорциональна массам двух задействованных объектов, и используя предыдущую интуицию о соотношении обратных квадратов силы между Землей и Луной, Ньютон смог сформулировать общий физический закон с помощью индукции.

Закон всемирного тяготения гласит, что каждая точечная масса притягивает любую другую точечную массу во Вселенной силой, направленной по прямой линии между центрами масс обеих точек, и эта сила пропорциональна массам объектов и обратно пропорциональна их разделению. Эта сила притяжения всегда направлена ​​внутрь, от одной точки к другой.Закон распространяется на все объекты большой или малой массы. Два больших объекта можно рассматривать как точечные массы, если расстояние между ними очень велико по сравнению с их размерами или если они сферически симметричны. Для этих случаев масса каждого объекта может быть представлена ​​как точечная масса, расположенная в его центре масс.

Хотя Ньютон смог сформулировать свой Закон всемирного тяготения и проверить его экспериментально, он мог только вычислить относительную гравитационную силу по сравнению с другой силой.2 [/ латекс]. Из-за величины [латекса] \ text {G} [/ latex] гравитационная сила очень мала, если не задействованы большие массы.

Силы на двух массах : Все массы притягиваются друг к другу. Сила пропорциональна массам и обратно пропорциональна квадрату расстояния.

Гравитационное притяжение сферических тел: однородная сфера

Теорема о оболочке утверждает, что сферически-симметричный объект влияет на другие объекты, как если бы вся его масса была сосредоточена в его центре.

Цели обучения

Сформулируйте теорему о оболочке для сферически-симметричных объектов

Основные выводы

Ключевые моменты
  • Поскольку сила является векторной величиной, векторная сумма всех частей оболочки вносит вклад в результирующую силу, и эта результирующая сила эквивалентна одному измерению силы, взятому из средней точки сферы или центра масс (COM).
  • Гравитационная сила, действующая на объект внутри полой сферической оболочки, равна нулю.
  • Гравитационная сила, действующая на объект с однородной сферической массой, линейно пропорциональна его расстоянию от центра масс сферы (COM). 2} [/ latex]

    Однако большинство объектов не являются точечными частицами.Чтобы найти гравитационную силу между трехмерными объектами, нужно рассматривать их как точки в пространстве. Для высокосимметричных форм, таких как сферы или сферические оболочки, найти эту точку просто.

    Теорема оболочек

    Исаак Ньютон доказал теорему оболочек, которая гласит:

    1. Сферически-симметричный объект воздействует на другие объекты гравитационно, как если бы вся его масса была сосредоточена в его центре,
    2. Если объект представляет собой сферически симметричную оболочку (т.е.е., полый шар), то чистая гравитационная сила на теле внутри него равна нулю.

    Поскольку сила является векторной величиной, векторная сумма всех частей оболочки / сферы вносит вклад в результирующую силу, и эта результирующая сила является эквивалентом одного измерения силы, взятого из средней точки сферы или центра масс (COM). . Таким образом, при определении силы тяжести, действующей на шар массой 10 кг, расстояние, измеренное от шара, берется от центра масс шара до центра масс Земли.

    Учитывая, что сферу можно представить как совокупность бесконечно тонких концентрических сферических оболочек (например, слоев луковицы), то можно показать, что следствием теоремы о оболочке является то, что сила, действующая на объект внутри твердой сферы зависит только от массы сферы внутри радиуса, на котором находится объект. Это потому, что оболочки с большим радиусом, чем тот, в котором находится объект, не , а не создают силу для объекта внутри них (утверждение 2 теоремы).

    При рассмотрении гравитационной силы, действующей на объект в точке внутри или вне однородного сферически-симметричного объекта радиуса [латекс] \ text {R} [/ latex], есть две простые и разные ситуации, которые должны быть Рассмотрены: случай полой сферической оболочки и случай твердой сферы с равномерно распределенной массой.

    Случай 1: полая сферическая оболочка

    Гравитационная сила, действующая сферически-симметричной оболочкой на точечную массу внутри ее, представляет собой векторную сумму гравитационных сил, действующих на каждую часть оболочки, и эта векторная сумма равна нулю.То есть масса [латекс] \ text {m} [/ latex] в пределах сферически симметричной оболочки массы [латекс] \ text {M} [/ latex] не будет ощущать чистой силы (утверждение 2 теоремы о оболочке ).

    Чистая гравитационная сила, которую сферическая оболочка из массы [латекс] \ text {M} [/ latex] оказывает на тело за пределами тела, представляет собой векторную сумму гравитационных сил, действующих на каждую часть оболочки на внешний объект, которые складываются в результирующую силу, действующую так, как будто масса [латекс] \ text {M} [/ latex] сосредоточена в точке в центре сферы (утверждение 1 теоремы о оболочке).

    Диаграмма, используемая в доказательстве теоремы о оболочке : На этой диаграмме показана геометрия, рассматриваемая при доказательстве теоремы о оболочке. В частности, в этом случае сферическая оболочка из массы [латекс] \ text {M} [/ latex] (левая часть рисунка) воздействует на массу [латекс] \ text {m} [/ latex] (правая часть рисунок) за его пределами. Цветом показана площадь поверхности тонкого среза сферы. (Примечание: доказательство теоремы здесь не приводится. Заинтересованные читатели могут продолжить изучение, используя источники, перечисленные в конце этой статьи.)

    Случай 2: твердая однородная сфера

    Вторая ситуация, которую мы рассмотрим, касается твердой однородной сферы массы [латекс] \ text {M} [/ latex] и радиуса [латекс] \ text {R} [/ latex], оказывающей силу на тело масса [латекс] \ text {m} [/ latex] с радиусом [латекс] \ text {d} [/ latex] внутри (то есть [латекс] \ text {d} <\ text {R }[/латекс]). Мы можем использовать результаты и следствия теоремы оболочек для анализа этого случая. Вкладом всех оболочек сферы с радиусом (или расстоянием) больше [latex] \ text {d} [/ latex] от центра масс сферы можно пренебречь (см. Выше следствие теоремы о оболочке).3 \ rho [/ латекс]

    ([латекс] \ rho [/ latex] — это массовая плотность сферы, и мы предполагаем, что она не зависит от радиуса. То есть масса сферы распределена равномерно.)

    Следовательно, объединяя два приведенных выше уравнения, получаем:

    [латекс] \ text {F} = \ frac {4} {3} \ pi \ text {Gm} \ rho \ text {d} [/ latex]

    , который показывает, что масса [латекс] \ text {m} [/ latex] испытывает силу, которая линейно пропорциональна его расстоянию, [latex] \ text {d} [/ latex], от центра масс сферы.

    Как и в случае полых сферических оболочек, чистая гравитационная сила, которую твердая сфера с равномерно распределенной массой [латекс] \ text {M} [/ latex] оказывает на тело за пределами от него, является векторной суммой гравитационные силы, действующие каждой оболочкой сферы на внешний объект. Результирующая чистая гравитационная сила действует так, как будто масса [латекс] \ text {M} [/ latex] сосредоточена в точке в центре сферы, которая является центром масс, или COM (утверждение 1 теоремы о оболочке).В более общем смысле, этот результат верен, даже если масса [латекс] \ text {M} [/ latex] равна , а не равномерно, но его плотность изменяется радиально (как в случае с планетами).

    Вес Земли

    Когда тела имеют пространственную протяженность, гравитационная сила вычисляется путем суммирования вкладов точечных масс, которые их составляют.

    Цели обучения

    Опишите, как рассчитывается сила тяжести для тел с пространственной протяженностью

    Основные выводы

    Ключевые моменты
    • Закон всемирного тяготения Ньютона гласит, что каждая точечная масса во Вселенной притягивает все остальные точечные массы с силой, которая прямо пропорциональна произведению их масс и обратно пропорциональна квадрату расстояния между ними.2 [/ latex], масса Земли рассчитывается как [латекс] 5,96 \ cdot 1024 [/ latex] кг, что позволяет рассчитать вес Земли при любом гравитационном поле.
    • Гравитация Земли может быть максимальной на границе ядро ​​/ мантия
    Ключевые термины
    • точка массы : Теоретическая точка с присвоенной ей массой.
    • вес : Сила, действующая на объект из-за гравитационного притяжения между ним и Землей (или каким-либо другим астрономическим объектом, который на него в первую очередь влияет).
    • гравитационная сила : очень дальнодействующая, но относительно слабая фундаментальная сила притяжения, которая действует между всеми частицами, имеющими массу; считается, что они связаны с гравитонами.

    Закон всемирного тяготения Ньютона гласит, что каждая точечная масса во Вселенной притягивает все остальные точечные массы с силой, которая прямо пропорциональна произведению их масс и обратно пропорциональна квадрату расстояния между ними.

    На современном языке закон гласит следующее: Каждая точечная масса притягивает каждую другую точечную массу силой, направленной вдоль линии, пересекающей обе точки .{2}} [/ латекс]

    где [latex] \ text {F} [/ latex] — сила между массами, [latex] \ text {G} [/ latex] — гравитационная постоянная, [latex] \ text {m} _1 [/ latex ] — первая масса, [latex] \ text {m} _2 [/ latex] — вторая масса, а [latex] \ text {r} [/ latex] — расстояние между центрами масс.

    Если рассматриваемые тела имеют пространственную протяженность (а не являются теоретическими точечными массами), то гравитационная сила между ними вычисляется путем суммирования вкладов условных точечных масс, составляющих тела.В пределе, когда составляющие точечные массы становятся «бесконечно малыми», это влечет за собой интегрирование силы (в векторной форме, см. Ниже) по всем двум телам.

    Таким образом можно показать, что объект со сферически-симметричным распределением массы оказывает такое же гравитационное притяжение на внешние тела, как если бы вся масса объекта была сосредоточена в точке в его центре.

    Для точек внутри сферически-симметричного распределения материи можно использовать теорему Ньютона Shell для определения силы тяжести.Теорема говорит нам, как различные части распределения массы влияют на гравитационную силу, измеренную в точке, расположенной на расстоянии [latex] \ text {r} _0 [/ latex] от центра распределения масс:

    1. Часть массы, расположенная по радиусам [латекс] \ text {r} <\ text {r} _0 [/ latex], вызывает ту же силу в [латексе] \ text {r} _0 [/ latex], что и если вся масса, заключенная в сфере радиуса [латекс] \ text {r} _0 [/ latex], была сосредоточена в центре распределения масс (как указано выше).
    2. Часть массы, расположенная по радиусам [латекс] \ text {r}> \ text {r} _0 [/ latex], не оказывает чистой гравитационной силы на расстоянии [latex] \ text {r} _0 [/ latex ] от центра. То есть отдельные гравитационные силы, действующие на элементы сферы снаружи, на точку [latex] \ text {r} _0 [/ latex], нейтрализуют друг друга.

    Как следствие, например, внутри оболочки одинаковой толщины и плотности нет чистого гравитационного ускорения где-либо в пределах полой сферы.Кроме того, внутри однородной сферы сила тяжести увеличивается линейно с расстоянием от центра; увеличение из-за дополнительной массы в 1,5 раза меньше уменьшения из-за большего расстояния от центра. Таким образом, если сферически-симметричное тело имеет однородное ядро ​​и однородную мантию с плотностью, меньшей, чем [latex] \ frac {2} {3} [/ latex], чем у ядра, то сила тяжести сначала уменьшается снаружи за пределы граница, и если сфера достаточно велика, дальше наружу сила тяжести снова увеличивается и в конечном итоге превышает силу тяжести на границе ядро ​​/ мантия.

    Гравитация Земли может быть максимальной на границе ядро ​​/ мантия, как показано на Рисунке 1:

    Гравитационное поле Земли : Диаграмма напряженности гравитационного поля внутри Земли.

    Universal Gravitation — The Physics Hypertextbook

    Обсуждение

    комета

    Исаак Ньютон родился в день Рождества 1642 года в деревне Вулсторп (недалеко от Грэнтэма), Линкольншир, Англия. В 1661 году он поступил в Тринити-колледж Кембриджского университета (примерно на полпути между Вулсторпом и Лондоном), где изучал математику.В 1665 году Черная чума достигла Англии, вынудив закрыть Тринити и отправив Ньютона обратно в Вулсторп на год или два. Именно в это время он сформулировал большинство своих важных вкладов в математику и физику, включая биномиальную теорему, дифференциальное исчисление, сложение векторов, законы движения, центростремительное ускорение, оптику и всемирное тяготение. По возвращении в Кембридж Ньютон стал профессором математики, а затем начал делать то, что профессора делают по сей день — преподавать и публиковать.Большинство статей, представленных Ньютоном для публикации, были посвящены оптике, особенно теории цвета. Затем, восемнадцать лет спустя, в 1684 году, Эдмонд Галлей пришел к Ньютону с проблемой, которую, по его мнению, Ньютон мог бы решить.

    Кометы — это астрономические объекты, которые можно увидеть без посторонней помощи всего месяц или около того. Они были серьезной проблемой для ранних астрономов, поскольку появлялись без предупреждения, какое-то время зависали в небе, а затем исчезали, чтобы их больше никогда не видели. Галлей изучал исторические записи появления комет, когда он заметил четыре кометы с почти одинаковой орбитой, разделенные во времени примерно на 76 лет.Он рассудил, что кометы 1456, 1531, 1607 и 1682 годов были наблюдениями одной кометы и что эта комета снова появится зимой 1758 года. Когда это произошло, как и предсказывалось, через шестнадцать лет после его смерти, она стала известна как комета Галлея. . Следует отметить, что Галлей не открыл комету, носящую его имя, он был как раз тем, кто идентифицировал ее как небесное тело с определенным периодом обращения по орбите вокруг Солнца. Комету Галлея, вероятно, видели еще на заре цивилизации, когда люди впервые посмотрели на небо и задались вопросом, как все это работает.Исторические записи из Индии, Китая и Японии свидетельствуют о его появлении еще в 240 г. до н. Э. (Одно появление не зарегистрировано). Последние его выступления были в 1833, 1909 и 1985 годах, а следующее — в 2061 году.

    Галлей также заметил, что комета описывает орбиту вокруг Солнца в соответствии с законами движения планет Кеплера; а именно, что орбита представляет собой эллипс (хотя и сильно вытянутый) с Солнцем в одном фокусе и что оно подчиняется гармоническому закону ( r 3 T 2 ), как если бы это была другая планета в Наша Солнечная система.В 1684 году Галлей спросил Ньютона, имеет ли он какое-нибудь представление о том, почему планеты и эта комета подчиняются законам Кеплера; то есть, если бы он знал природу ответственной силы. Ньютон ответил, что он действительно решил эту проблему и «многое другое», относящееся к механике, восемнадцатью годами ранее, но никому об этом не сказал. Затем он начал рыться в поисках своих записей о годах чумы, но не смог их найти. Галлей убедил Ньютона собрать все, что он когда-либо знал по механике, и предложил оплатить затраты, чтобы его идеи могли быть опубликованы.

    В 1687 году, после восемнадцати месяцев безостановочной работы, Ньютон опубликовал Philosophiæ Naturalis Principia Mathematica ( The Mathematical Principia Natural Philosophy ). Вероятно, самая важная книга по физике и, возможно, величайшая книга во всей науке, она почти всегда известна просто как «Принципы », . Он содержит суть концепций, представленных в главах по механике каждого последующего учебника физики, в том числе и этого.Вероятно, единственное важное понятие, которое он упускает, — это энергия, но все остальное присутствует: сила, масса, ускорение, инерция, импульс, вес, сложение векторов, движение снаряда, круговое движение, движение спутника, гравитация, приливные силы, прецессия равноденствий. ….

    ОТСУТСТВУЕТ ТЕКСТ

    В 1684 году D r Галлей приехал навестить его в Кембридже, после того, как они некоторое время были вместе, D r спросил его, какой, по его мнению, будет кривая, которую будут описывать планеты, предполагающие силу притяжения к Солнце обратно пропорционально квадрату их расстояния от него.S r Исаак немедленно ответил, что это будет эллипсис, доктор был поражен радостью и изумлен, спросил его, откуда он это узнал, почему говорит, что я его вычислил, после чего доктор Галлей без дальнейших промедлений попросил его вычислить: r Исаак поискал среди своих бумаг, но не нашел их, но пообещал возобновить их; а затем отправить ему

    Абрахам де Муавр, 1727

    ОТСУТСТВУЕТ ТЕКСТ

    De motu corporum in gyrum (О движении тел по орбите) — это (предполагаемое) название рукописи Исаака Ньютона, отправленной Эдмонду Галлею в ноябре 1684 года.

    закон

    The Principia содержит в себе объединение земного и небесного тяготения. Ускорение свободного падения, описанное Галилеем, и законы движения планет, наблюдаемые Кеплером, — это разные аспекты одного и того же. Нет земного тяготения для Земли и небесного тяготения для планет, а скорее универсального тяготения для всего.

    1. Каждый объект во Вселенной притягивает любой другой объект во Вселенной с помощью гравитационной силы.
    2. Величина гравитационной силы между двумя объектами составляет…
      1. прямо пропорционально произведению их масс и
      2. обратно пропорционально квадрату расстояния между их центрами

    Закон Ньютона работает, поскольку мы живем во вселенной с тремя пространственными измерениями. По мере того, как гравитация распространяется в космос, она распространяется все тоньше и тоньше, покрывая площадь, которая увеличивается в квадрате расстояния от источника.Если бы пространство не было трехмерным, закон Ньютона не работал бы.

    Хотя пространство кажется трехмерным, нет очевидной причины, почему это должно быть. Некоторые пока еще спекулятивные теории предполагают, что могут быть дополнительные пространственные измерения. Причина, по которой мы их не видели, в том, что они довольно плотно свернулись. Если они существуют, должно быть возможно найти отклонения силы тяжести от закона обратных квадратов Ньютона на чрезвычайно малых расстояниях. Проверить эти отклонения довольно сложно.Лучшие эксперименты (по состоянию на 2001 год) показывают, что закон обратных квадратов сохраняется до 218 мкм (2,18 × 10 –4 м). Поскольку предполагается, что размер этих скрытых размеров составляет порядка 10 — 35 м, нам еще предстоит пройти долгий путь.

    луна

    Расстояние между Землей и Луной примерно в шестьдесят раз больше, чем радиус Земли. Ускорение свободного падения на этом расстоянии составляет 1 3600 ускорение свободного падения на поверхности Земли.

    яблоко

    Исаак Ньютон поступил в Тринити-колледж Кембриджского университета в 1661 году. Он получил степень бакалавра гуманитарных наук в 1665 году, когда Великая чума охватила Лондон. Кембриджский университет был закрыт в качестве меры предосторожности, и Ньютон сбежал на ферму своей семьи в Линкольншире, в 90 км к северу. Летом 1666 года Ньютон начал работу над своей теорией всемирного тяготения. Чуть более двадцати лет спустя окончательная теория была обнародована как часть его большого фолианта Philosophiæ Naturalis Principia Mathematica ( Mathematical Principia Natural Philosophy ).Яблоки не обсуждались.

    Перенесемся в 1726 год. Сэр Исаак Ньютон был легендой, которая приближалась к концу своей жизни. Он обедал с другом, Уильямом Стьюкли, после чего они сидели в саду и о многом говорили. Ньютону было 83 года, когда он вспомнил событие, произошедшее 60 годами ранее. Это история, как рассказывает Стьюкли (с использованием его оригинального написания, заглавных букв и знаков препинания).

    15 апреля 1726 года я нанес визит сэру Исааку в его квартиру в зданиях Орбелс, Кенсингтон: пообщался с ним … appletrees, только он и я.он сказал мне, что среди других рассуждений он был точно в той же ситуации, когда раньше ему приходило в голову понятие гравитации. «Почему это яблоко всегда должно опускаться перпендикулярно земле», — подумал он про себя из-за падения яблока, когда он сидел в задумчивом настроении: «Почему оно не должно идти вбок или вверх? а постоянно? к центру Земли? Разумеется, причина в том, что Земля втягивает его. В материи должна быть сила притяжения. И сумма притягивающей силы в материи Земли должна быть в центре Земли, а не на какой-либо стороне земли.Поэтому падает ли это яблоко перпендикулярно или по направлению к центру. если материя таким образом влечет материю; он должен быть пропорционален его количеству. поэтому яблоко рисует Землю, так же как Земля рисует яблоко ».

    Уильям Стьюкли, 1752

    Другой вариант яблочной истории был записан помощником Ньютона на Королевском монетном дворе (а также его племянником в законе) Джоном Кондуиттом.

    В год [1666] он снова ушел из Кембриджа в связи с чумой своей матери [в] Линкольншире, и пока он размышлял в саду, ему пришла в голову мысль, что та же сила тяжести (w ch заставил яблоко упасть с дерева на землю) не ограничивался определенным расстоянием от Земли, но эта сила должна распространяться намного дальше, чем обычно думали — почему не так высоко, как Луна сказал он себе, и если да, то должен влиять на ее движение и, возможно, удерживать ее на орбите, после чего он упал на расчет… и нашел, что это полностью согласуется с его Теорией —

    Джон Кондуитт, ок.1728

    Сам Ньютон никогда ничего не писал о яблоках. Его больше интересовало движение Луны как средство проверки его теории.

    В том же году [1666] я начал думать о гравитации, распространяющейся на шар Луны, и обнаружил, как оценить силу, с которой шар, вращающийся внутри сферы, давит на поверхность сферы, исходя из правила Кеплера. периодичность планет, находящихся в полуторной пропорции их расстояний от центров их сфер, я сделал вывод, что силы, удерживающие планеты в их орбах, должны быть обратно пропорциональны квадратам их расстояний от центров, вокруг которых они вращаются: и таким образом сравнил силу, необходимую для удержания Луны в ее орбите, с силой тяжести на поверхности Земли, и нашел, что они ответили довольно близко.Все это было в два года чумы — 1665 и 1666, потому что в те дни я был в расцвете сил для изобретательства и думал о математике и философии больше, чем когда-либо с тех пор.

    Исаак Ньютон, ок. 1715

    Когда Ньютона спросили, как он открыл закон всемирного тяготения, он ответил:

    , если я оказал или публичную услугу таким образом, то это только из-за промышленности и терпеливой мысли.

    Исаак Ньютон, 1692

    формула

    Force

    F г = — Gm 1 m 2
    r 2

    Поле

    Напряженность гравитационного поля (ускорение свободного падения) ☞ Напряженность гравитационного поля для черных дыр была рассчитана на поверхности горизонта событий (радиус Шварцшильда).Гравитация на горизонте событий может приближаться к бесконечности. Также обратите внимание, что напряженность гравитационного поля на горизонте событий уменьшается с увеличением массы черной дыры.
    объект Масса (кг) радиус (км) г (м / с 2 ) г (г)
    Солнце 1,99 × 10 30 696 000 270 28
    Меркурий 3.30 × 10 23 2,440 3,7 0,38
    Венера 4,87 × 10 24 6 050 8,9 0,90
    Земля 5,97 × 10 24 6,380 9,8 1,0
    Луна 7,36 × 10 22 1,740 1.6 0,17
    Марс 6,42 × 10 23 3 400 3,7 0,38
    Юпитер 1,90 × 10 27 71 500 25 2,5
    Сатурн 5,69 × 10 26 60 300 10 1,1
    Уран 8.68 × 10 25 25 600 8,9 0,90
    Нептун 1,02 × 10 26 24 800 11 1,1
    Плутон 1,31 × 10 22 1,180 0,63 0,064
    белый карлик ~ 1 солнечная масса ~ 1 Радиус Земли ~ 3 000 000 ~ 300 000
    нейтронная звезда 2 ~ 3 массы Солнца ~ 10 ~ 10 13 ~ 10 12
    звездная черная дыра> 3 массы Солнца> 9 <5 × 10 12 <5 × 10 11
    сверхмассивная черная дыра 10 5 ~ 10 9 массы Солнца 10 5 ~ 10 9 10 8 ~ 10 4 10 7 ~ 10 3

    постоянная

    Эксперимент Кавендиша

    Великая пирамида настолько массивна, что отвес не будет свисать прямо вниз, когда рядом с пирамидой, а будет качаться в сторону конструкции.Ср. Tompkins, Secrets of the Great Pyramids, стр. 84-85, где Томпкинс, обсуждая измерения, сделанные Пьяцци Смит, пишет: «Чтобы получить правильную широту Великой пирамиды, не отклоняя его отвес от перпендикуляра из-за притяжения огромная часть пирамиды, Смит проводил свои наблюдения с вершины; там сила тяжести пирамиды будет направлена ​​прямо вниз ». Томпкинс, Питер. Тайны Великой пирамиды (Нью-Йорк: Харпер Коллинз, 1971).

    критики

    Действие на расстоянии.Ответ Ньютона на эту критику был в основном: «Мне все равно. Теория работает».

    Rationem vero harum gravitatis proprietatum ex phænomenis nondum potui deducere, & hypotheses non fingo …. Et Satis est quod gravitas revera existat, & agat secundum leges a nobis expositas, & ad corporum cælestium & maris nostri motus omnes sufficiat. Я не смог обнаружить причину этих свойств гравитации на основе явлений, и Я не выдвигаю никаких гипотез ….И для нас достаточно того, что гравитация действительно существует и действует согласно законам, которые мы объяснили, и в значительной степени служит для объяснения всех движений небесных тел и наших морей.

    сверх того…

    • Кто-то изобрел гравитационное поле. Единицы: Н / кг или м / с 2
    • Счастливая эквивалентность инертной и гравитационной масс.
    • Без сомнения, Ньютон думал, что Бог говорил с ним, но Библия не упоминает закон всемирного тяготения.
    • Ньютон сошел с ума на пару лет, вероятно, из-за отравления ртутью.
    • Он больше думал о своем библейском анализе, чем о своем физическом анализе.
    • Ньютон был назначен мастером монетного двора — по сути, должность патронажа, чтобы вознаградить его за достижения в области физики. В то время как там он использовал зубчатые монеты, чтобы предотвратить их «стрижку» или «бритье», что в то время было серьезной проблемой в Англии.
    • Ньютон придумал слово гравитация от gravitas , латинского слова, обозначающего тяжесть, серьезность или власть.Латинское слово для обозначения веса — pondus , что дает нам английскую единицу веса — фунт (но, что интересно, не глагол фунт или фунт, в котором содержатся бездомные животные).

    СДЕЛАТЬ НОВЫЙ РАЗРЕЗ ДЛЯ ГРАВИТАЦИОННОГО ПОЛЯ

    дополнительные размеры

    Цитата (измененная) Учителя физики, которая будет перефразирована. «Существенным ингредиентом для образования черных дыр на LHC (Большом адронном коллайдере) является наличие дополнительных измерений. Черная дыра — это область интенсивного гравитационного поля, создающего условия, противоречащие тому, что мы наблюдаем в отношении гравитационных сил в повседневной жизни. Мир.Наличие дополнительных измерений гарантирует дополнительную силу гравитации, необходимую для создания черных дыр. Когда протоны сталкиваются в LHC, они подходят так близко друг к другу, что по сути «видят» дополнительные измерения (где сильная гравитация), и возможно образование черной дыры. Если это так, то дополнительные размеры должны составлять ~ 10 −14 м. «

    Трехмерное пространство

    n-мерное пространство

    Гравитационная сила в n > трехмерном пространстве увеличивается быстрее на малых расстояниях, чем в трехмерном пространстве.Предполагается, что размеры больше 3 малы и, следовательно, выявляются только при небольших расстояниях между ними.

    Эпизод 401: Закон всемирного тяготения Ньютона

    .

    Закон всемирного тяготения Ньютона

    Земля и космос | Силы и движение

    Эпизод 401: Закон всемирного тяготения Ньютона

    Урок для 16-19

    • Время активности 65 минут
    • Уровень Продвинутый

    Этот эпизод знакомит с законом всемирного тяготения Ньютона для точечных и сферических масс, а также дает студентам возможность попрактиковаться в расчетах силы между объектами.Обсуждается значение закона обратных квадратов .

    Краткое содержание урока

    • Обсуждение: Введение в закон всемирного тяготения Ньютона (5 минут)
    • Обсуждение: Закон всемирного тяготения Ньютона: F гравитация = G M m r 2
    • (10 минут)
    • Рабочие примеры: Использование F силы тяжести = G × м 1 × м 2 r 2 (25 минут)
    • Вопросы студентов: Больше практики с F гравитация = G м 1 м 2 r 2 (20 минут)
    Обсуждение: Введение в закон всемирного тяготения Ньютона

    Вот несколько вопросов и ответов, которые ведут к закону всемирного тяготения Ньютона.

    Что вызывает лишний вес, который испытывает каждый ученик? (гравитационное притяжение Землей.)

    Что влияет на силу притяжения Земли на вас? Зачем вам на Луне весить другое? (Ваша масса и ее масса.)

    Если Земля обрушивается на вас, то что еще должно происходить согласно 3-му закону Ньютона? (Вы должны подтягиваться на Земле с силой, равной вашему весу.)

    Что происходит с силой притяжения Земли, когда вы удаляетесь от нее? (Он становится слабее — большинство студентов догадываются об этом правильно из неверного предположения, что в космосе космонавты невесомые!)

    Итак, в целом сила зависит от массы Земли и вас самих и ослабевает с расстоянием.Все это воплощено в законе всемирного тяготения Ньютона

    .
    Обсуждение: закон всемирного тяготения Ньютона

    Представьте уравнение, которое представляет закон всемирного тяготения Ньютона.

    F Плотность = G м 1 м 2 r 2

    F гравитация сила притяжения в ньютонах

    м 1 , м 2 — взаимодействующие массы в килограммах

    r — разделение двух масс в метрах

    G известна как универсальная гравитационная постоянная (, а не , следует путать с маленькими г).Он устанавливает силу гравитационного взаимодействия в том смысле, что если бы оно было удвоено, то же самое произошло бы со всеми гравитационными силами.

    G = 6,67 × 10 -11 Н м 2 кг -2

    Покажите, как можно вычислить единицы, переставив исходное уравнение.

    Этот закон применяется к точечным массам, но сферические массы можно рассматривать так, как если бы они были точечными массами, вся их масса сосредоточена в их центре.

    Эта сила всегда привлекательна .В некоторых текстах вы увидите знак минус в уравнении, так что F сила тяжести = — G m 1 m 2 r 2 . Этот знак минус используется исключительно для обозначения силы притяжения (это пережиток более правильного, но далеко выходящего за рамки учебной программы векторного уравнения, выражающего закон всемирного тяготения Ньютона). Проще всего рассчитать величину силы, используя

    F = G m 1 m 2 r 2 , и направление задается тем фактом, что сила всегда притягивает.

    Каждый объект с массой во Вселенной притягивает друг друга в соответствии с этим законом. Но реальный размер силы становится очень малым для объектов, находящихся очень далеко. Например, Солнце примерно в миллион раз массивнее Земли, но из-за того, что оно находится так далеко, притяжение Солнца на нас затмевается притяжением со стороны Земли (которое примерно в 1650 раз больше). По мере того, как расстояние между двумя объектами увеличивается, расстояние 2 увеличивается еще больше, резко.Гравитационная сила уменьшится во столько же раз (так как расстояние 2 фигурирует в знаменателе уравнения). Это пример закона обратных квадратов , названного так потому, что сила притяжения изменяется обратно пропорционально квадрату разделения.

    Рабочие примеры: Использование
    F = G m 1 m 2 r 2

    Вы можете проработать эти примеры или поставить их в качестве задания для своих учеников, если вы чувствуете, что они смогут их решить.

    Эпизод 401-1: Рабочие примеры; Силы гравитации (Word, 29 КБ)

    Вопросы студентов: больше практики с
    F = G m 1 m 2 r 2

    Эпизод 401-2: Закон тяготения Ньютона (Word, 53 КБ)

    Закон всемирного тяготения Ньютона

    Закон всемирного тяготения был кульминацией многолетних усилий Исаака Ньютона, направленных на понимание силы притяжения между массами.

    Согласно легенде, когда Ньютон увидел яблоко, падающее с дерева, он пришел к выводу, что сила должна притягивать яблоко к Земле. Если бы эта сила могла действовать на вершине дерева, она могла бы действовать на еще больших расстояниях. В то время он изучал орбиты Луны и планет и в конечном итоге сформулировал закон всемирного тяготения, объясняющий их движение.

    Закон всемирного тяготения Ньютона гласит, что каждая частица во Вселенной притягивает каждую другую частицу с силой, пропорциональной произведению их масс и обратно пропорциональной квадрату расстояния между ними.

    В этом видео показано, как экспериментально измерить ускорение свободного падения и сравнить его с теоретическим значением из уравнения, определяющего силу тяжести.

    Прежде чем углубляться в эксперимент, давайте рассмотрим принципы, лежащие в основе Закона всемирного тяготения. Гравитационная сила Земли на Луне равна по величине и противоположна по направлению силе Луны на Земле. Эта сила FG действует вдоль линии, соединяющей их центры масс.

    Согласно закону всемирного тяготения, FG равно G — всемирной гравитационной постоянной, умноженной на произведение двух масс, деленное на квадрат r, который представляет собой расстояние между их центрами масс.

    С помощью этого выражения можно вычислить гравитационную силу, которую Земля оказывает на объект на любом расстоянии, в том числе вблизи или на его поверхности. В случае падения яблока с дерева, предположим, что масса яблока равна m, масса Земли равна mE, ​​а радиус равен rE.

    Второй закон движения Ньютона утверждает, что сила равна массе, умноженной на ускорение. Если мы объединим это уравнение, примененное к яблоку, с законом всемирного тяготения, мы сможем сократить массу m яблока с обеих сторон. В этом контексте ускорение обычно обозначается буквой g

    Итак, сила тяжести на яблоке задается Законом всемирного тяготения, но, исходя из второго закона движения, эта сила также может быть выражена в мг. Как мы видели ранее на примере Земли и Луны, сила Земли на яблоке такая же, как сила яблока на Земле.Но почему мы видим только яблоко, падающее на Землю? Почему мы не видим, как Земля движется к яблоку?

    Если мы посмотрим на второй закон движения Ньютона, мы можем изменить его так, чтобы показать, что ускорение равно силе, деленной на массу. То есть для данной силы ускорение обратно пропорционально массе. Поскольку Земля намного массивнее яблока, ускорение Земли по направлению к яблоку незначительно и практически необнаружимо. Вот почему яблоко падает с дерева.

    Возвращаясь к уравнению гравитации для g, поскольку все значения в правой части — универсальная гравитационная постоянная, масса Земли и радиус Земли — известны для объекта, близкого к поверхности Земли, Величина g также является стандартной величиной, которая составляет 9,8 метра на секунду в квадрате.

    Однако это значение можно рассчитать экспериментально, просто сбросив шар с известной высоты и применив кинематические уравнения. И мы продемонстрируем, как это сделать, в следующих разделах.

    В этом эксперименте используются металлический шар, измерительная ручка, один датчик, на котором мяч будет подвешен, другой датчик, на который мяч приземлится, один таймер, подключенный к обоим датчикам, один зажим и одна стойка-штанга. Сначала с помощью зажима прикрепите шариковый датчик к стержню на высоте не менее 0,5 метра над поверхностью стола. Затем поместите второй датчик непосредственно под первым датчиком.

    Затем измерьте расстояние между верхним и нижним датчиками. Расстояние следует измерять относительно низа мяча.

    Теперь отпустите мяч из датчика, чтобы он упал на нижний датчик, и запишите время.

    Повторите эту процедуру пять раз, а затем вычислите среднее время падения.

    Из кинематического видео в этой коллекции мы знаем, что эта формула описывает положение при одномерном движении объекта с постоянным ускорением.

    Поскольку мы имеем дело с гравитацией Земли, ускорение в данном случае — это ускорение свободного падения, или g. И начальная скорость равна нулю, так как мяч перед падением находился в состоянии покоя.Итак, если мы переместим начальную позицию на другую сторону уравнения, левая сторона станет y минус y0, что является не чем иным, как d — расстоянием между начальной и конечной точками измерения. Теперь мы можем переписать уравнение для g.

    Для этого эксперимента d составляло 0,72 метра, а среднее время свободного падения составляло 0,382 секунды. В результате экспериментальное ускорение свободного падения составляет 9,9 метра в секунду в квадрате. Эксперимент и теория отличаются только примерно на 1%, что указывает на то, что закон всемирного тяготения Ньютона является очень хорошим описанием гравитационного притяжения.

    Универсальный закон всемирного тяготения участвует в расчетах, выполняемых различными отраслями техники.

    Раздел машиностроения, называемый статикой, занимается изучением сил, действующих на стационарные объекты, например мосты. Инженеры, проектирующие мосты, используют статику, и особенно уравнение F = mg, на протяжении всей своей работы для анализа нагрузок на конструкции.

    В миссии НАСА по гравитационному картированию используются два идентичных спутника: один ведущий, а другой — замыкающий спутник, вращающиеся вместе с Землей.Когда ведущий спутник проходит над ледяной шапкой или другим скоплением массы, он ускоряется из-за относительно большей силы притяжения. Задний спутник испытывает такое же ускорение, когда проходит над той же областью.

    Система измерения дальности измеряет, как и где изменяется расстояние между ними, предоставляя информацию о распределении массовых концентраций вокруг Земли.

    Вы только что посмотрели введение JoVE в закон всемирного тяготения Ньютона. Теперь вы должны знать, как определить силу тяжести между двумя массами, и понять, как рассчитать ускорение, вызванное силой тяжести на поверхности Земли.Спасибо за просмотр!

    7.2 Закон всемирного тяготения Ньютона и общая теория относительности Эйнштейна

    Концепции, связанные с законом всемирного тяготения Ньютона

    Сэр Исаак Ньютон был первым ученым, который точно определил силу гравитации и показал, что она может объяснить как падающие тела, так и астрономические движения. См. Рисунок 7.8. Но Ньютон был не первым, кто подозревал, что одна и та же сила вызывает и наш вес, и движение планет.Его предшественник, Галилео Галилей, утверждал, что падающие тела и движения планет имеют одну и ту же причину. Некоторые современники Ньютона, такие как Роберт Гук, Кристофер Рен и Эдмунд Галлей, также достигли некоторого прогресса в понимании гравитации. Но Ньютон был первым, кто предложил точную математическую форму и использовал ее, чтобы показать, что движение небесных тел должно иметь конические сечения — окружности, эллипсы, параболы и гиперболы. Это теоретическое предсказание стало большим триумфом.В течение некоторого времени было известно, что луны, планеты и кометы следуют по таким путям, но никто не смог предложить объяснения механизма, который заставил их следовать по этим путям, а не по другим.

    Рис. 7.8 В популярной легенде о том, что Ньютон внезапно открыл закон всемирного тяготения, когда яблоко упало с дерева и ударило его по голове, есть доля правды. Более вероятно, что он шел через фруктовый сад и задавался вопросом, почему все яблоки падают в одном направлении с одинаковым ускорением.Этому придается большое значение, потому что универсальный закон тяготения Ньютона и его законы движения ответили на очень старые вопросы о природе и оказали огромную поддержку понятию лежащей в основе простоты и единства в природе. Ученые по-прежнему ожидают, что простота, лежащая в основе, возникнет в результате их постоянных исследований в природе.

    Гравитационная сила относительно проста. Это всегда привлекательно, и это зависит только от вовлеченных масс и расстояния между ними. Выраженный современным языком, универсальный закон всемирного тяготения Ньютона гласит, что каждый объект во Вселенной притягивает любой другой объект с силой, которая направлена ​​вдоль соединяющей их линии.Сила прямо пропорциональна произведению их масс и обратно пропорциональна квадрату расстояния между ними. Это притяжение проиллюстрировано на рис. 7.9.

    Рис. 7.9 Гравитационное притяжение происходит вдоль линии, соединяющей центры масс (ЦМ) двух тел. Величина силы, действующей на каждое тело, одинакова, что соответствует третьему закону Ньютона (действие-противодействие).

    Для двух тел массой m и M с расстоянием r между их центрами масс уравнение универсального закона всемирного тяготения Ньютона составляет

    , где F — величина силы тяжести, а G — коэффициент пропорциональности, называемый гравитационной постоянной. G — универсальная постоянная, означающая, что она считается одинаковой во всей Вселенной. Экспериментально определено, что G = 6,673 × 10-11 Н⋅м2 / кг2G = 6,673 × 10-11 Н⋅м2 / кг2.

    Если человек имеет массу 60,0 кг, какой будет сила гравитационного притяжения на нем у поверхности Земли? G приведено выше, масса Земли M составляет 5,97 × 10 24 кг, а радиус r Земли составляет 6,38 × 10 6 м. Помещение этих значений в универсальный закон всемирного тяготения Ньютона дает

    F = GmMr2 = (6.673 × 10-11 Н · м2 кг2) ((60,0 кг) (5,97 · 1024 кг) (6,38 · 106 м) 2) = 584 НФ = GmMr2 = (6,673 · 10-11 Н · м2 кг2) ((60,0 кг) ( 5,97 × 1024 кг) (6,38 × 106 м) 2) = 584 Н

    Этот результат можно проверить соотношением: F = mg = (60 кг) (9,8 м / с2) = 588 NF = mg = (60 кг) (9,8 м / с2) = 588 Н

    Вы можете помнить, что g , ускорение свободного падения, является еще одной важной константой, связанной с гравитацией. Подставляя г вместо на в уравнении для второго закона движения Ньютона, мы получаем F = mgF = mg. Комбинируя это с уравнением всемирного тяготения, получаем

    . Сокращение массы m с обеих сторон уравнения и заполнение значений гравитационной постоянной, массы и радиуса Земли дает значение г, , которое может выглядит знакомо.

    g = GMr2 = (6,67 × 10-11 Н⋅м2 кг2) (5,98 × 1024 кг (6,38 × 106 м) 2) = 9,80 м / с2g = GMr2 = (6,67 × 10-11 Н⋅м2 кг2) (5,98 × 1024 кг (6,38 × 106 м) 2) = 9,80 м / с2

    Это хороший момент, чтобы вспомнить разницу между массой и весом. Масса — это количество вещества в объекте; Вес — это сила притяжения между массой двух объектов. Вес может меняться, потому что г различаются на каждой луне и планете. Масса объекта м не меняется, но его вес м г может.

    Виртуальная физика

    Гравитация и орбиты

    Переместите Солнце, Землю, Луну и космическую станцию ​​в этой симуляции, чтобы увидеть, как это влияет на их гравитационные силы и орбитальные траектории. Визуализируйте размеры и расстояния между разными небесными телами. Отключите гравитацию и посмотрите, что будет без нее!

    Проверка захвата

    Почему Луна не движется по плавному кругу вокруг Солнца?

    1. На Луну не действует гравитационное поле Солнца.
    2. На Луну не действует гравитационное поле Земли.
    3. На Луну действуют гравитационные поля Земли и Солнца, которые всегда аддитивны.
    4. На Луну действуют гравитационные поля Земли и Солнца, которые иногда складываются, а иногда противоположны.

    Snap Lab

    Эксперимент на вынос: падающие предметы

    В этом упражнении вы изучите влияние массы и сопротивления воздуха на ускорение падающих предметов.Сделайте прогнозы (гипотезы) об исходе этого эксперимента. Запишите их, чтобы потом сравнить с результатами.

    Материалы

    • Четыре листа бумаги 8-1 / 2 × 118-1 / 2 × 11 дюймов

    Процедура

    • Возьмите четыре одинаковых листа бумаги.
      • Скомкайте один в маленький шар.
      • Оставьте один без мятого.
      • Возьмите два других и скомкайте их вместе, чтобы получился шар, в два раза превышающий массу другого скомканного шара.
      • Теперь сравните, какой шарик из бумаги приземляется первым при одновременном падении с одной и той же высоты.
        1. Сравните скомканный шарик из одной бумаги со скомканным шариком из двух листов.
        2. Сравните скомканный бумажный шарик с немятой бумагой.

    Проверка захвата

    Почему одни объекты падают у поверхности земли быстрее других, если вся масса одинаково притягивается силой тяжести?

    1. Некоторые объекты падают быстрее из-за сопротивления воздуха, которое действует в направлении движения объекта и оказывает большую силу на объекты с меньшей площадью поверхности.
    2. Некоторые объекты падают быстрее из-за сопротивления воздуха, которое действует в направлении, противоположном движению объекта, и оказывает большую силу на объекты с меньшей площадью поверхности.
    3. Некоторые объекты падают быстрее из-за сопротивления воздуха, которое действует в направлении движения объекта и оказывает большую силу на объекты с большей площадью поверхности.
    4. Некоторые объекты падают быстрее из-за сопротивления воздуха, которое действует в направлении, противоположном движению объекта, и оказывает большую силу на объекты с большей площадью поверхности.

    Третий закон Кеплера можно вывести из закона всемирного тяготения Ньютона. Применение второго закона Ньютона к угловому движению дает выражение для центростремительной силы, которое можно приравнять к выражению для силы в уравнении всемирного тяготения. Этим выражением можно манипулировать, чтобы получить уравнение третьего закона Кеплера. Ранее мы видели, что выражение r 3 / T 2 является константой для спутников, вращающихся вокруг одного и того же массивного объекта.Вывод третьего закона Кеплера из закона всемирного тяготения Ньютона и второго закона движения Ньютона дает эту константу:

    , где M — масса центрального тела, вокруг которого вращаются спутники (например, Солнце в нашей солнечной системе). Полезность этого уравнения станет очевидной позже.

    Универсальная гравитационная постоянная G определена экспериментально. Это определение впервые было дано точно в 1798 году английским ученым Генри Кавендишем (1731–1810), более чем через 100 лет после того, как Ньютон опубликовал свой универсальный закон всемирного тяготения.Измерение G очень простое и важное, поскольку оно определяет силу одной из четырех сил в природе. Эксперимент Кавендиша был очень трудным, потому что он измерил крошечное гравитационное притяжение между двумя массами обычного размера (максимум десятки килограммов) с помощью прибора, подобного показанному на рис. 7.11. Примечательно, что его стоимость для G отличается от современной стоимости менее чем на 1%.

    Рис. 7.11 Кавендиш использовал подобное устройство для измерения гравитационного притяжения между двумя подвешенными сферами ( m ) и двумя сферами на подставке ( M ), наблюдая за величиной скручивания (скручивания), создаваемого в волокне.Расстояние между массами можно варьировать, чтобы проверить зависимость силы от расстояния. Современные эксперименты этого типа продолжают исследовать гравитацию.

    Общая теория относительности Эйнштейна

    Общая теория относительности Эйнштейна объяснила некоторые интересные свойства гравитации, не охваченные теорией Ньютона. Эйнштейн основал свою теорию на постулате о том, что ускорение и гравитация имеют одинаковый эффект и их нельзя отличить друг от друга. Он пришел к выводу, что свет должен падать как в гравитационном поле, так и в ускоряющейся системе отсчета.На рис. 7.12 показан этот эффект (сильно преувеличенный) в ускоряющемся лифте. На рис. 7.12 (a) лифт ускоряется вверх в невесомости. На рис. 7.12 (b) комната не ускоряется, но подвержена действию силы тяжести. Эффект на свет одинаков: в обоих случаях он «падает» вниз. Человек в лифте не может определить, ускоряется ли лифт в условиях невесомости или он неподвижен и подвержен действию силы тяжести. Таким образом, гравитация влияет на путь света, даже если мы думаем, что гравитация действует между массами, в то время как фотоны безмассовые.

    Рис. 7.12 (a) Луч света выходит из фонарика в лифте, ускоряющемся вверх. Поскольку лифт движется вверх в течение времени, необходимого свету, чтобы достичь стены, луч падает ниже, чем если бы лифт не был ускорен. (b) Гравитация должна иметь такое же влияние на свет, поскольку невозможно определить, ускоряется ли лифт вверх или неподвижен и на него действует сила тяжести.

    Общая теория относительности Эйнштейна получила свое первое подтверждение в 1919 году, когда во время солнечного затмения наблюдали свет звезд, проходящий около Солнца.(См. Рис. 7.13.) Во время затмения небо темнеет, и мы можем ненадолго увидеть звезды. Те, кто находится на линии прямой видимости, ближайшей к солнцу, должны иметь смещение в их видимом положении. Этот сдвиг не только наблюдался, но и хорошо согласовывался с предсказаниями Эйнштейна в пределах экспериментальных неопределенностей. Это открытие произвело фурор в науке и общественности. Эйнштейн теперь стал народным героем, а также очень великим ученым. Искривление света материей эквивалентно искривлению самого пространства, когда свет движется по кривой.Это еще одно радикальное изменение в нашем представлении о пространстве и времени. Это также еще одна связь, что на любую частицу с массой или энергией (например, безмассовые фотоны) действует сила тяжести.

    Рис. 7.13 На этой схеме показано, как свет, проходящий рядом с массивным телом, таким как солнце, изгибается к нему. Тогда кажется, что свет, достигающий Земли, исходит из разных мест, отличных от известных положений исходящих звезд. Не только наблюдался этот эффект, но и величина изгиба была в точности тем, что Эйнштейн предсказал в своей общей теории относительности.

    Обобщая два взгляда на гравитацию, Ньютон представил гравитацию как перетягивание каната вдоль линии, соединяющей любые два объекта во Вселенной. Напротив, Эйнштейн представлял гравитацию как искривление пространства-времени массой.

    Безграничная физика

    Гравитационный зонд НАСА B

    Миссия НАСА Gravity Probe B (GP-B) подтвердила два ключевых предсказания, основанных на общей теории относительности Альберта Эйнштейна. Зонд, показанный на рис. 7.14, был запущен в 2004 году. На нем было четыре сверхточных гироскопа, предназначенных для измерения двух эффектов, предполагаемых теорией Эйнштейна:

    • Геодезический эффект, который представляет собой искривление пространства и времени гравитационным полем массивного тела (в данном случае Земли)
    • Эффект перетаскивания кадра, который представляет собой величину, на которую вращающийся объект тянет за собой пространство и время при вращении.

    Рисунок 7.14 Художественная концепция космического корабля Gravity Probe B на орбите вокруг Земли. (кредит: NASA / MSFC)

    Оба эффекта были измерены с беспрецедентной точностью. Это было сделано путем наведения гироскопов на одиночную звезду, вращаясь вокруг Земли по полярной орбите. Согласно теории относительности, гироскопы испытали очень небольшие, но измеримые изменения направления вращения, вызванные притяжением Земли.

    Главный исследователь предложил представить себе Землю, вращающуюся в меде.Когда Земля вращается, она увлекает за собой пространство и время, как окружающее море меда.

    Проверка захвата

    Согласно общей теории относительности, гравитационное поле искривляет свет. При чем здесь время и пространство?

    1. Гравитация не влияет на пространственно-временной континуум, а гравитация влияет только на движение света.
    2. Пространственно-временной континуум искажен гравитацией, и гравитация не влияет на движение света.
    3. Гравитация не влияет ни на пространственно-временной континуум, ни на движение света.
    4. Пространственно-временной континуум искажен гравитацией, и гравитация влияет на движение света.

    13,1 Закон всемирного тяготения Ньютона — University Physics Volume 1

    Притяжение между галактиками
    Найдите ускорение нашей галактики, Млечного Пути, по отношению к ближайшей галактике сопоставимого размера, галактике Андромеды (рис.13.5). Приблизительная масса каждой галактики составляет 800 миллиардов солнечных масс (солнечная масса — это масса нашего Солнца), и их разделяет 2,5 миллиона световых лет. (Обратите внимание, что масса Андромеды не так хорошо известна, но считается, что она немного больше, чем наша галактика.) Каждая галактика имеет диаметр примерно 100000 световых лет (1 световой год = 9,5 × 1015 м) (1 световой год = 9,5 × 1015м).

    Рис. 13.5. Галактики гравитационно взаимодействуют на огромных расстояниях. Галактика Андромеды — ближайшая к Млечному Пути спиральная галактика, и в конечном итоге они столкнутся.(кредит: Борис Штромар)

    Стратегия
    Как и в предыдущем примере, мы используем закон всемирного тяготения Ньютона для определения силы между ними, а затем используем второй закон Ньютона для определения ускорения Млечного Пути. Мы можем считать галактики точечными массами, поскольку их размеры примерно в 25 раз меньше расстояния между ними. Масса Солнца (см. Приложение D) составляет 2,0 × 1030 кг2,0 × 1030 кг, а световой год — это расстояние, которое свет проходит за один год, 9,5 × 1015 м9,5 × 1015 м.
    Решение
    Величина силы равна F12 = Gm1m2r2 = (6.67 × 10–11 Н · м2 / кг2) [(800 × 109) (2,0 × 1030 кг)] 2 [(2,5 × 106) (9,5 × 1015 м)] 2 = 3,0 × 1029 Н. F12 = Gm1m2r2 = (6,67 × 10− 11 Н · м2 / кг2) [(800 × 109) (2,0 × 1030 кг)] 2 [(2,5 × 106) (9,5 × 1015 м)] 2 = 3,0 × 1029 Н.

    Ускорение Млечного Пути

    a = Fm = 3,0 × 1029 Н (800 × 109) (2,0 × 1030 кг) = 1,9 × 10-13 м / с 2. a = Fm = 3,0 × 1029 Н (800 × 109) (2,0 × 1030 кг) = 1,9 × 10-13 м / с s2.
    Значение
    Не кажется ли такое значение ускорения поразительно малым? Если они начнут из состояния покоя, то они будут ускоряться прямо навстречу друг другу, «сталкиваясь» в своем центре масс.Прикинем, когда это должно произойти. Начальное ускорение составляет ~ 10-13 м / с2 ~ 10-13 м / с2, поэтому, используя v = atv = at, мы видим, что каждой галактике потребуется ~ 1013 ~ 1013 с, чтобы достичь скорости 1,0 м / с, и они будет всего на ~ 0,5 × 1013 м ~ 0,5 × 1013 м ближе. Это на девять порядков меньше первоначального расстояния между ними. На самом деле такие движения редко бывают простыми. Эти две галактики, а также около 50 других меньших галактик, все гравитационно связаны в нашем локальном скоплении. Наше локальное скопление гравитационно связано с другими скоплениями в так называемом сверхскоплении.Все это — часть великого космического танца, являющегося результатом гравитации, как показано на рис. 13.6.

    Рис. 13.6 На основании результатов этого примера и того, что астрономы наблюдали в других частях Вселенной, наша галактика столкнется с галактикой Андромеды примерно через 4 миллиарда лет.